Show the following polynomial is Irreducible over the given ring












6












$begingroup$


Studying for a qualifying exam and this practice problem flat out has me stumped. I wish to show that the polynomial $(y+8)^2x^3 - x^2 + (y+7)(y+8) - y - 12$ is irreducible over $mathbb{Q}[x, y]$. My thought was to use Eisenstein's for $mathbb{Q}[x][y]$ and $mathbb{Q}[y][x]$, however both variations haven't yielded a solution. For example, in $mathbb{Q}[y][x]$, the prime ideal would need to contain $-1$, but this cannot happen. Writing this polynomial as a polynomial in
$mathbb{Q}[x][y]$ yields the polynomial $(x^3+1)y^2 + (16x^3 + 14)y + 64x^3 - x^2 + 44$, but haven't been able to find a prime ideal to satisfy Eisenstein's. Any idea would be appreciated.










share|cite|improve this question









$endgroup$












  • $begingroup$
    Figured it out, in case anyone is working on it.
    $endgroup$
    – Travis62
    Dec 21 '18 at 19:38








  • 1




    $begingroup$
    Might I suggest that you post an answer to your own question? This would remove it from the "unanswered" tab.
    $endgroup$
    – Pierre-Guy Plamondon
    Dec 23 '18 at 16:29
















6












$begingroup$


Studying for a qualifying exam and this practice problem flat out has me stumped. I wish to show that the polynomial $(y+8)^2x^3 - x^2 + (y+7)(y+8) - y - 12$ is irreducible over $mathbb{Q}[x, y]$. My thought was to use Eisenstein's for $mathbb{Q}[x][y]$ and $mathbb{Q}[y][x]$, however both variations haven't yielded a solution. For example, in $mathbb{Q}[y][x]$, the prime ideal would need to contain $-1$, but this cannot happen. Writing this polynomial as a polynomial in
$mathbb{Q}[x][y]$ yields the polynomial $(x^3+1)y^2 + (16x^3 + 14)y + 64x^3 - x^2 + 44$, but haven't been able to find a prime ideal to satisfy Eisenstein's. Any idea would be appreciated.










share|cite|improve this question









$endgroup$












  • $begingroup$
    Figured it out, in case anyone is working on it.
    $endgroup$
    – Travis62
    Dec 21 '18 at 19:38








  • 1




    $begingroup$
    Might I suggest that you post an answer to your own question? This would remove it from the "unanswered" tab.
    $endgroup$
    – Pierre-Guy Plamondon
    Dec 23 '18 at 16:29














6












6








6





$begingroup$


Studying for a qualifying exam and this practice problem flat out has me stumped. I wish to show that the polynomial $(y+8)^2x^3 - x^2 + (y+7)(y+8) - y - 12$ is irreducible over $mathbb{Q}[x, y]$. My thought was to use Eisenstein's for $mathbb{Q}[x][y]$ and $mathbb{Q}[y][x]$, however both variations haven't yielded a solution. For example, in $mathbb{Q}[y][x]$, the prime ideal would need to contain $-1$, but this cannot happen. Writing this polynomial as a polynomial in
$mathbb{Q}[x][y]$ yields the polynomial $(x^3+1)y^2 + (16x^3 + 14)y + 64x^3 - x^2 + 44$, but haven't been able to find a prime ideal to satisfy Eisenstein's. Any idea would be appreciated.










share|cite|improve this question









$endgroup$




Studying for a qualifying exam and this practice problem flat out has me stumped. I wish to show that the polynomial $(y+8)^2x^3 - x^2 + (y+7)(y+8) - y - 12$ is irreducible over $mathbb{Q}[x, y]$. My thought was to use Eisenstein's for $mathbb{Q}[x][y]$ and $mathbb{Q}[y][x]$, however both variations haven't yielded a solution. For example, in $mathbb{Q}[y][x]$, the prime ideal would need to contain $-1$, but this cannot happen. Writing this polynomial as a polynomial in
$mathbb{Q}[x][y]$ yields the polynomial $(x^3+1)y^2 + (16x^3 + 14)y + 64x^3 - x^2 + 44$, but haven't been able to find a prime ideal to satisfy Eisenstein's. Any idea would be appreciated.







ring-theory irreducible-polynomials polynomial-rings






share|cite|improve this question













share|cite|improve this question











share|cite|improve this question




share|cite|improve this question










asked Dec 21 '18 at 1:54









Travis62Travis62

714




714












  • $begingroup$
    Figured it out, in case anyone is working on it.
    $endgroup$
    – Travis62
    Dec 21 '18 at 19:38








  • 1




    $begingroup$
    Might I suggest that you post an answer to your own question? This would remove it from the "unanswered" tab.
    $endgroup$
    – Pierre-Guy Plamondon
    Dec 23 '18 at 16:29


















  • $begingroup$
    Figured it out, in case anyone is working on it.
    $endgroup$
    – Travis62
    Dec 21 '18 at 19:38








  • 1




    $begingroup$
    Might I suggest that you post an answer to your own question? This would remove it from the "unanswered" tab.
    $endgroup$
    – Pierre-Guy Plamondon
    Dec 23 '18 at 16:29
















$begingroup$
Figured it out, in case anyone is working on it.
$endgroup$
– Travis62
Dec 21 '18 at 19:38






$begingroup$
Figured it out, in case anyone is working on it.
$endgroup$
– Travis62
Dec 21 '18 at 19:38






1




1




$begingroup$
Might I suggest that you post an answer to your own question? This would remove it from the "unanswered" tab.
$endgroup$
– Pierre-Guy Plamondon
Dec 23 '18 at 16:29




$begingroup$
Might I suggest that you post an answer to your own question? This would remove it from the "unanswered" tab.
$endgroup$
– Pierre-Guy Plamondon
Dec 23 '18 at 16:29










1 Answer
1






active

oldest

votes


















3












$begingroup$

Consider the homomorphism $y to -7$ from $mathbb{Q}[x, y] to mathbb{Q}[x]$. Then the image is a proper prime ideal of $mathbb{Q}[x]$ and one can show then that the preimage must also be a prime ideal in $mathbb{Q}[x, y]$.






share|cite|improve this answer









$endgroup$














    Your Answer





    StackExchange.ifUsing("editor", function () {
    return StackExchange.using("mathjaxEditing", function () {
    StackExchange.MarkdownEditor.creationCallbacks.add(function (editor, postfix) {
    StackExchange.mathjaxEditing.prepareWmdForMathJax(editor, postfix, [["$", "$"], ["\\(","\\)"]]);
    });
    });
    }, "mathjax-editing");

    StackExchange.ready(function() {
    var channelOptions = {
    tags: "".split(" "),
    id: "69"
    };
    initTagRenderer("".split(" "), "".split(" "), channelOptions);

    StackExchange.using("externalEditor", function() {
    // Have to fire editor after snippets, if snippets enabled
    if (StackExchange.settings.snippets.snippetsEnabled) {
    StackExchange.using("snippets", function() {
    createEditor();
    });
    }
    else {
    createEditor();
    }
    });

    function createEditor() {
    StackExchange.prepareEditor({
    heartbeatType: 'answer',
    autoActivateHeartbeat: false,
    convertImagesToLinks: true,
    noModals: true,
    showLowRepImageUploadWarning: true,
    reputationToPostImages: 10,
    bindNavPrevention: true,
    postfix: "",
    imageUploader: {
    brandingHtml: "Powered by u003ca class="icon-imgur-white" href="https://imgur.com/"u003eu003c/au003e",
    contentPolicyHtml: "User contributions licensed under u003ca href="https://creativecommons.org/licenses/by-sa/3.0/"u003ecc by-sa 3.0 with attribution requiredu003c/au003e u003ca href="https://stackoverflow.com/legal/content-policy"u003e(content policy)u003c/au003e",
    allowUrls: true
    },
    noCode: true, onDemand: true,
    discardSelector: ".discard-answer"
    ,immediatelyShowMarkdownHelp:true
    });


    }
    });














    draft saved

    draft discarded


















    StackExchange.ready(
    function () {
    StackExchange.openid.initPostLogin('.new-post-login', 'https%3a%2f%2fmath.stackexchange.com%2fquestions%2f3048114%2fshow-the-following-polynomial-is-irreducible-over-the-given-ring%23new-answer', 'question_page');
    }
    );

    Post as a guest















    Required, but never shown

























    1 Answer
    1






    active

    oldest

    votes








    1 Answer
    1






    active

    oldest

    votes









    active

    oldest

    votes






    active

    oldest

    votes









    3












    $begingroup$

    Consider the homomorphism $y to -7$ from $mathbb{Q}[x, y] to mathbb{Q}[x]$. Then the image is a proper prime ideal of $mathbb{Q}[x]$ and one can show then that the preimage must also be a prime ideal in $mathbb{Q}[x, y]$.






    share|cite|improve this answer









    $endgroup$


















      3












      $begingroup$

      Consider the homomorphism $y to -7$ from $mathbb{Q}[x, y] to mathbb{Q}[x]$. Then the image is a proper prime ideal of $mathbb{Q}[x]$ and one can show then that the preimage must also be a prime ideal in $mathbb{Q}[x, y]$.






      share|cite|improve this answer









      $endgroup$
















        3












        3








        3





        $begingroup$

        Consider the homomorphism $y to -7$ from $mathbb{Q}[x, y] to mathbb{Q}[x]$. Then the image is a proper prime ideal of $mathbb{Q}[x]$ and one can show then that the preimage must also be a prime ideal in $mathbb{Q}[x, y]$.






        share|cite|improve this answer









        $endgroup$



        Consider the homomorphism $y to -7$ from $mathbb{Q}[x, y] to mathbb{Q}[x]$. Then the image is a proper prime ideal of $mathbb{Q}[x]$ and one can show then that the preimage must also be a prime ideal in $mathbb{Q}[x, y]$.







        share|cite|improve this answer












        share|cite|improve this answer



        share|cite|improve this answer










        answered Dec 27 '18 at 16:06









        Travis62Travis62

        714




        714






























            draft saved

            draft discarded




















































            Thanks for contributing an answer to Mathematics Stack Exchange!


            • Please be sure to answer the question. Provide details and share your research!

            But avoid



            • Asking for help, clarification, or responding to other answers.

            • Making statements based on opinion; back them up with references or personal experience.


            Use MathJax to format equations. MathJax reference.


            To learn more, see our tips on writing great answers.




            draft saved


            draft discarded














            StackExchange.ready(
            function () {
            StackExchange.openid.initPostLogin('.new-post-login', 'https%3a%2f%2fmath.stackexchange.com%2fquestions%2f3048114%2fshow-the-following-polynomial-is-irreducible-over-the-given-ring%23new-answer', 'question_page');
            }
            );

            Post as a guest















            Required, but never shown





















































            Required, but never shown














            Required, but never shown












            Required, but never shown







            Required, but never shown

































            Required, but never shown














            Required, but never shown












            Required, but never shown







            Required, but never shown







            Popular posts from this blog

            Plaza Victoria

            In PowerPoint, is there a keyboard shortcut for bulleted / numbered list?

            How to put 3 figures in Latex with 2 figures side by side and 1 below these side by side images but in...